Tải bản đầy đủ (.docx) (7 trang)

Mot So Bat Dang Thuc Thuong Dung

Bạn đang xem bản rút gọn của tài liệu. Xem và tải ngay bản đầy đủ của tài liệu tại đây (118.92 KB, 7 trang )

<span class='text_page_counter'>(1)</span>BẤT ĐẲNG THỨC – GIÁ TRỊ LỚN NHẤT VÀ GIÁ TRỊ NHỎ NHẤT Bất đẳng thức Cauchy Với a1, a2, a3, ..., an là những số dương, n ≥ 2, ta luôn có a1 + a 2 + a 3 +...... + a n ³ n. n. a1a 2a 3...a n. Đẳng thức xảy ra khi và chỉ khi a1 = a2 = a3 = ... = an. Chứng minh bất đẳng thức Cauchy: Có 3 cách chứng minh như sau Cách 1: Dùng phương pháp quy nạp toán học a1 + a 2 ³ 2 Bước 1: Thử với n = 2 thì. a1a 2 Û ( a1 -. a 2 )2 ³ 0. (đúng). Dấu “=” xảy ra khi a1 = a2. Bước 2: Giả sử như biểu thức (*) đúng khi n = k ta có a1 + a 2 +... + a k ³ k. k. a1a 2 ...a k. (1) Dấu “=” xảy ra khi a1 = a2 = ... = ak. Bước 3: Ta cần chứng minh (*) sễ đúng với n = k + 1. Xét (k + 1) số không âm a1, a2, ..., ak+1. Nếu trong các số này có số lớn nhất và nhỏ nhất, ta có quyền sắp xếp sao cho ak là một trong các số nhỏ nhất và ak+1 là một trong các số lớn nhất. Nếu chúng không có số lớn nhất và nhỏ nhất thì chúng sẽ bằng nhau ta không cần sắp xếp. μ=. a1 + a 2 +... + a k+1 k +1 (2) thì khi đó ak+1 ≥ μ ≥ ak. (2’). Đặt Đặt bk = ak + ak+1 – μ suy ra ak + ak+1 = bk + μ (3) Từ (2) và (3) suy ra (k + 1)μ = a1 + a2 + ... + ak–1 + bk + μ hay kμ = a1 + a2 + ... + ak–1 + bk. Þ μ=. a1 + a 2 +... + a k- 1 + b k k. Áp dụng giả thuyết (1) ta được μ³. k. a1a 2 ...a k- 1b k Û μ k ³ a1a 2 ...a k- 1 (a k + a k+1 - μ). Û μ k +1 ³ a1a 2 ...a k- 1 (a k + a k +1 - μ)μ (4). Bây giờ ta thấy (ak + ak+1 – μ)μ – akak+1 = ak(μ – ak+1) + (ak+1 – μ)μ = (ak+1 – μ)(μ – ak) ≥ 0 do điều kiện (2’) Từ đó suy ra (ak + ak+1 – μ)μ ≥ akak+1 (5) Û μ³. k. a a ...a. 1 2 k +1 (6) Thay (5) vào (4) ta được μk+1 ≥ a1a2...ak+1 Từ (6) suy ra điều phải chứng minh ở bước 3. Dấu “=” xảy ra khi a1 = a2 = ... = ak–1 = bk = μ và (ak+1 – μ)(μ – ak) = 0. Nói cách khác dấu “=” xảy ra khi a1 = a2 = ... = ak–1 = ak = ak+1. Bước 4: Theo nguyên lý quy nạp thì bất đẳng thức (*) luôn đúng với điều kiện ở đề bài. Cách 2: Dùng tính đơn điệu của hàm số Xét hàm số f(x) = ex–1 – x. Đạo hàm f’(x) = ex–1 – 1. f’(x) = 0 khi và chỉ khi x = 1. f’’(x) = ex–1 > 0 nên f’’(1) > 0. Hàm số này nghịch biến trên (–∞; 0) và đồng biến trên (0; +∞). min[f(x)] = f(1) = 0 nên f(x) ≥ f(1) = 0 với mọi x Hay ex–1 ≥ x (1) với mọi x. Xét một dãy số thực không âm a1, a2, ..., an có trung bình cộng là μ. Nếu μ = 0 thì a1 = a2 = ... = an = 0 dẫn đến (*) hiển nhiên đúng. Nếu μ > 0 ta có:.

<span class='text_page_counter'>(2)</span> a1. a2. an. a1. a2. an. -1 -1 -1 - 1+ - 1+...+ - 1 a1 a 2 a n μ μ × ... £ e μ e μ ...e μ = e μ μ μ μ (2) a1 a2 an a1 + a 2 +... + a n - 1 + - 1 +... + - 1 = - n =n- n =0 μ μ μ μ Mà. (2) Û. a1.a 2 ...a n μ. n. £ e0 = 1 Û μ ³. n. a1.a 2 ...a n. (3). Vậy ta có điều phải chứng minh. Dấu “=” xảy ra khi a1 = a2 = ... = an = μ. Cách 3: Chứng minh của Cauchy a1 + a 2 ³ 2 Xét trường hợp n = 2 thì. a1a 2 Û ( a1 -. a 2 )2 ³ 0. (đúng). Dấu “=” xảy ra khi a1 = a2. Giả sử trường hợp n = 2k = b đúng. Khi đó xét n = 2k+1 = 2b ta có a1 + a 2 +... + a 2b a1 + a 2 +... + a b a b+1 + a b+2 +...a 2b 1 b = + ³ ( a1a 2 ...a b + b a b+1a b+2 ...a 2b ) 2b 2b 2b 2 (2) 1 b ( a1a 2 ...a b + b a b+1a b+2 ...a 2b ) ³ b a1a 2 ...a 2b = 2b a1a 2 ...a 2b Áp dụng cho hai số ta được 2 (3) a1 + a 2 +... + a 2b 2b ³ a1a 2 ...a 2b 2b Từ (2) và (3) suy ra. Do đó bất đẳng thức luôn đúng với mọi n = 2k, k là số nguyên dương. Nếu n không phải là 2k, thì tồn tại giá trị k sao cho 2k > n. Ta đặt m = 2k – n. Xét n số không âm a1, a2, ..., an có trung bình cộng là μ. Xét tiếp m số an+1, an+2, ..., an+m sao cho an+1 = an+2 = ... = an+m = μ. Như vậy ta áp dụng bất thức n + m = 2k số như sau a1 + a 2 +... + a n + a n+1 + a n +2 +... + a n+m ³ (n + m) n+m a1a 2 ...a n a n+1a n+2 ...a n+m Û (n + m)μ ³ (n + m) n+m a1a 2 ...aμn. m. Û μ n+m ³ a1a 2 ...a n μ m Û μ n ³ a1a 2 ...a n. Từ đó suy ra điều phải chứng minh. Bất đẳng thức Bunyakovsky – Cauchy – Schwarz (B.C.S) Với mọi a1, a2, ..., an, b1, b2, ..., bn ta luôn có (a1b1 + a 2b 2 +... + a n b n )2 £ (a12 + a 22 +... + a 2n )(b12 + b 22 +... + b 2n ) a1 a 2 a 3 a = = = ....... = n bn Đẳng thức xảy ra khi và chỉ khi b1 b 2 b3. Đặc biệt: nếu a1, a2, ..., an, b1, b2, ..., bn > 0 b12 b 22 b 2 (b + b 2 +... + b n ) 2 + +... + n ³ 1 a1 a 2 an a1 + a 2 +... + a n b1 b 2 b3 b = = = ....... = n an Đẳng thức xảy ra khi và chỉ khi a1 a 2 a 3. Chứng minh bất đẳng thức B.C.S 2 2 2 2 2 Xét n = 2: (a1b1 + a 2 b 2 ) £ (a1 + a 2 )(b1 + b 2 ). Û a12 b12 + 2a1a 2 b1b 2 + a 22 b 22 £ (a12 b12 + a12 b 22 + a 22 b12 + a 22b 22 ) Û 0 £ a12 b22 - 2a1a 2 b1b2 + a 22 b12 Û 0 £ (a1b 2 - a 2 b1 ) 2 (Đúng).

<span class='text_page_counter'>(3)</span> a1 a 2 = b b2 1 Đẳng thức xảy ra khi và chỉ khi a1b2 = a2b1 hay 2 2 2 2 2 2 2 Giả sử n = k ta có (a1b1 + a 2 b2 +... + a k b k ) £ (a1 + a 2 +... + a k )(b1 + b 2 +... + b k ) a1 a 2 a 3 a = = = ....... = k bk Đẳng thức xảy ra khi và chỉ khi b1 b 2 b3. →. a1b1 + a 2 b2 +... + a k bk £ a12 + a 22 +... + a k2 . b12 + b22 +... + b 2k. Mặt khác:. a1b1 + a 2 b 2 +... + a k b k + a k+1b k +1 £ a1b1 + a 2b 2 +... + a k b k + a k +1b k+1. a 2 + a 2 +... + a 2. b 2 + b 2 +... + b 2k. 2 k và b = 1 2 Đặt a = 1 Áp dụng B.C.S cho bộ bốn số a, b, |ak+1|, |bk+1|:. ab + |ak+1||bk+1| ≤ →. a 2 + a 2k+1 . b 2 + b2k+1 = a12 + a 22 +... + a 2k+1 . b12 + b 22 +... + b2k+1. a1b1 + a 2 b2 +... + a k bk + a k +1b k +1 £ a12 + a 22 +... + a k2+1 . b12 + b 22 +... + b k2+1 a a1 a 2 a 3 a = = = ....... = k = k+1 b1 b 2 b3 b k b k +1. Đẳng thức xảy ra khi và chỉ khi Nên n = k + 1 vẫn đúng Theo nguyên lý quy nạp bất đẳng thức đúng với mọi n ≥ 2 BÀI TẬP MINH HỌA. A=. a 2. +. b 2. +. c. 1+ b 1+c 1+a Bài toán 1. Cho a, b, c > 0 và a + b + c = 3. Tìm giá trị nhỏ nhất Hướng dẫn giải Nhận xét do tính đối xứng của (a; b; c) nên đẳng thức có thể xảy ra khi a = b = c = 1. a 2. =a-. Ta có: 1 + b mà 1 + b² ≥ 2b a 2. ³ a-. 2. ab 2 1 + b2. ab 2. → 1+ b Chứng minh tương tự và cộng vế theo vế: 1 A ≥ a + b + c – 2 (ab + bc + ca). (1) Mặt khác 2ab ≤ a² + b²; 2bc ≤ b² + c²; 2ca ≤ c² + a² → 2ab + 2bc + 2ca ≤ 2(a² + b² + c²) → ab + bc + ca ≤ a² + b² + c² → 3(ab + bc + ca) ≤ (a + b + c)² = 9 → ab + bc + ca ≤ 3 (2) 3 Vậy từ (1) và (2) ta được A ≥ 2. Đẳng thức xảy ra khi và chỉ khi. ïìï a = b = c í ïïî a + b + c = 3. hay a = b = c = 1. Bài toán 2. Cho a, b, c > 0 và a + b + c = 3. Tìm giá trị nhỏ nhất Hướng dẫn giải. A=. 1+ a 1 + b2. +. 1+ b 1 + c2. +. 1+ c 1+a 2.

<span class='text_page_counter'>(4)</span> 1+a 2. = (a +1) -. b 2 (a +1). b(a +1) 2. ³ (a +1) -. 2. 1+ b Ta có 1 + b Chứng minh tương tự và cộng vế theo vế ta được 3+. a + b + c ab + bc + ca 2 2. A≥ Mà 3(ab + bc + ca) ≤ (a + b + c)² = 9 → ab + bc + ca ≤ 3 Từ đó suy ra A ≥ 3 Đẳng thức xảy ra khi và chỉ khi a = b = c = 1 Bài toán 3. Chứng minh rằng với mọi số dương a, b, c , d ta luôn có a3 2. a +b. 2. +. b3 2. b +c. 2. +. c3 2. c +d. 2. +. d3 2. d +a. 2. a +b +c +d 2. ³. Hướng dẫn giải a3 2 2 Ta có a + b. =ab3. 2. Tương tự: b + c. 2. ab 2 a 2 + b2 ³ b-. b 2. ³ a-. c3. c ; 2. 2. c +d a. 3. 2. 2. +. ³ c-. 2. b. 3. 2. 2. +. d3. d ; 2. 2. d +a c. 3. 2. 2. +. 2. ³ d-. d3 2. 2. ³. b +c c +d d +a Cộng vế theo vế ta được: a + b Bài toán 4. Chứng minh rằng với mọi số dương a, b, c ,ta luôn có a3 2. a + ab + b. 2. +. b3 2. b + bc + c. + 2. c3 2. c + ca + a. 2. ³. a 2. a +b +c +d 2. a + b +c 3. Hướng dẫn giải Ta có a3 a 2 + ab + b 2. =a-. ab ( a + b) a 2 + ab + b 2. ³ a-. ab ( a + b ) a +b =a3ab 3. Chứng minh tương tự b3 b 2 + bc + c 2. ³ b-. b +c ; 3. c3 c 2 + ca + a 2. ³ c-. c +a 3. Cộng vế theo vế được điều phải chứng minh Bài toán 5. CHo a, b, c là độ dài ba cạnh của một tam giác với chu vi 2p. Chứng minh rằng: abc a. (p – a)(p – b)(p – c) ≤ 8 æ ö 1 1 1 1 1 1÷ + + ³ 2ç + + ÷ ç ÷ ç èa b c ø b. p - a p - b p - c. Hướng dẫn giải a. Áp dụng bất đẳng thức Cauchy với 2 số dương p - a +p - b c = (1) 2 2 p - b +p - c a (p - b)(p - c) £ = (2) 2 2 p - c +p - a b (p - c)(p - a) £ = (3) 2 2 (p - a)(p - b) £. Nhân vế theo vế (1), (2), (3) ta được đpcm. b. Áp dụng bất đẳng thức ta có.

<span class='text_page_counter'>(5)</span> 1 1 4 4 + ³ = p- a p- b p- a +p- b c 1 1 4 4 + ³ = p- b p- c p- b +p- c a 1 1 4 4 + ³ = p- c p - a p - c +p - a b. Cộng vế theo vế ta được đpcm BÀI TẬP VẬN DỤNG Bài 1. Cho a, b, c > 0 và a + b + c = 3 a2. Chứng minh 1 + 2b. + 3. b2. c2 + ³ 1 1 + 2c3 1 + 2a 3 1 3. 3. Bài 2. Cho a, b, c > 0. Chứng minh a + b + abc Bài 3. Cho a, b, c, d > 0 và a + b + c + d = 4. a2. b2. 1 + c2d. +. c2. +. 1 3. 3. b + c + abc. +. 1 3. 3. c + a + abc. +£. 1 abc. d2. ³ 2 1 + d 2a 1 + a 2 b ab bc ca a +b +c + + £ 2 Bài 4. Cho a, b, c > 0. Chứng minh a + b b + c c + a 2 Chứng minh 1 + b c. +. +. Bài 5. Cho 0 ≤ x ≤ 1, 0 ≤ y ≤ 2. Chứng minh (1 – x)(2 – y)(4x – y) ≤ 4. Đẳng thức xảy ra khi nào. 1 1 1 + + = 20 Bài 6. Cho x, y, z là các số dương thỏa mãn x y z 1 1 1 + + £5 Chứng minh rằng x + y + 2z y + z + 2x z + x + 2y. Bài 7. Cho x, y > 0 và x² + y² = 1. Tìm giá trị nhỏ nhất A = x³ + y³. Bài 8. Cho x, y, z > 0 và x² + y² + z² = 3. Tìm giá trị nhỏ nhất A = x4 + y4 + z4. Bài 9. Cho x, y > 0; 0 < a < b; a, b nguyên và xa + ya = 1. Tìm giá trị nhỏ nhất A = xb + yb. Bài 10. Cho x, y, z > 0, và x4 + y4 + z4 = 3. Tìm giá trị nhỏ nhất A = x6 + y6 + z6. Bài 11. Cho x, y > 0 và 3x² + 4y² = 1. Tìm giá trị nhỏ nhất A = 5x³ + 6y³ Bài 12. Cho x, y > 0 và 7x² + 8y² = 1. Tìm giá trị nhỏ nhất A = 9x³ + 10y³ 1 1 1 + + ³ 2 Bài 13. Cho ba số dương a, b, c thỏa mãn điều kiện 1 + a 1 + b 1 + c . Chứng minh rằng abc. ≤ 0,125. Gợi ý: 1/(1 + a) = b/(1 + b) + c/(1 + c) Bài 14. Cho ba số a, b, c > 0. Chứng minh (1 + a)(1 + b)(1 + c) ≥ (1 + Bài 15. Cho ba số a, b, c > 0 thỏa mãn ab + bc + ca = abc. b 2 + 2a 2 c2 + 2b2 a 2 + 2c2 + + ³ ab bc ca. 3. abc )³. 3. Chứng minh: Bài 16. Cho ba số a, b, c bất kỳ. Chứng minh rằng (ab + bc + ca)² ≥ 3abc(a + b + c) 1 1 1 1 + + + ³ 3 Bài 17. Cho a, b, c, d là bốn số dương thỏa mãn điều kiện 1 + a 1 + b 1 + c 1 + d. Chứng minh rằng abc ≤ 1/81 1 1 x 2 + y 2 + + ³ 2( x + y) x y Bài 18. Chứng minh rằng với mọi x, y > 0 ta có.

<span class='text_page_counter'>(6)</span> Bài 19. Cho x, y là những số dương thỏa mãn điều kiện x + y = 1. Hãy tìm giá trị nhỏ nhất của P = xy +. 1 xy. biểu thức Bài 20. Cho x, y, z thỏa mãn x² + y² + z² = 1 tìm giá trị lớn nhất, và nhỏ nhất của P = x + y + z + xy + yz + zx. Bài 21. Cho x, y > 0 và x + y = 1. Hãy tìm giá trị lớn nhất và nhỏ nhất của biểu thức: P = 3x + 9y. Bất đẳng thức vector Ví dụ 1. Cho x, y, z > 0. 2 2 2 2 2 2 Chứng minh rằng x + xy + y + y + yz + z + z + zx + x ³ Hướng dẫn giải Trong mặt phẳng Oxy gọi ba vectơ. 3(x + y + z).  y 3y  z 3z  x 3x a = (x + ; ) ; b = (y + ; ) ; c = (z + ; ) 2 2 2 2 2 2    3 3 Þ a + b + c = ( (x + y + z); (x + y + z)) 2 2       a + b + c ³ a +b +c. Theo bất đẳng thức vectơ: Þ. x 2 + xy + y 2 + y 2 + yz + z 2 + z 2 + zx + x 2 ³. 3(x + y + z). BÀI TẬP VẬN DỤNG Bài 1. Cho ba số x, y, z > 0 thỏa mãn x + y + z ≤ 1 x2 +. Chứng minh rằng: Bài 2. Chứng minh rằng. 1 x. 2. + y2 +. 1 y. 2. + z2 +. 1 z2. ³. 82. 2 2 2 2 2 2 a. (a + b) + (b + d) £ a + b + c + d 2 2 2 2 2 2 b. (a + c) + b + (a - c) + b ³ 2 a + b. Bài 3. Cho ba số x, y, z > 0, chứng minh rằng x + y + z £ 3 x + y + z Bài 4. Chứng minh với mọi số thực x, y, z ta có xyz(x + y + z) ≤ x4 + y4 + z4. Định lí Lagrange Nếu hàm số y = f(x) liên tục trên đoạn [a; b] và có đạo hàm trên khoảng (a; b) thì tồn tại f '(c) . f (b)  f (a) b a. một điểm c thuộc (a; b) sao cho → Hệ số góc tiếp tuyến tại (c; f(c)) bằng với hệ số góc của cát tuyến AB. Bài toán 1. Tìm c trong công thức Lagrange với y = f(x) = x² – x trên đoạn [1; 5]. Hướng dẫn giải Hàm số liên tục trên đoạn [1; 5] và có đạo hàm trên khoảng (1; 5) nên theo Lagrange ta có f '(c) =. f (b) - f (a) b- a. <=> 2c – 1 = 5 <=> c = 3 a- b a a- b < ln < b b Bài toán 2. Chứng minh rằng nếu 0 < b < a thì a. Hướng dẫn giải Xét hàm số f(x) = ln x trên đoạn [b; a].

<span class='text_page_counter'>(7)</span> 1 Đạo hàm f’(x) = x xác định trên (b; a) ln a - ln b a a- b $c Î (a; b) : = f ¢(c) Þ ln = a- b b c Theo Lagrange ta có a- b a- b a- b < < c b Mà 0 < b < c < a suy ra a a- b a a- b < ln < b b Vậy a 1 n 1 < ln < n- 1 n- 1 Bài toán 3. Chứng minh rằng nếu số nguyên n > 1 thì n 1 1 1 1 1 + +... + < ln n <1 + +... + n 2 n- 1 Suy ra: 2 3. Hướng dẫn giải Xét hàm số f(x) = ln x trên đoạn [n – 1, n] 1 Đạo hàm f’(x) = x xác định trên (n – 1, n). Theo Lagrange: ln n - ln(n- 1) n 1 = f '(c) Þ ln = n - (n - 1) n- 1 c 1 1 1 < < Mà 0 < n – 1 < c < n → n c n - 1 1 n 1 < ln < n- 1 n- 1 → n $c Î (n - 1; n) :. n = 2: 1/2 < ln 2 < 1 n = 3: 1/3 < ln 3 – ln 2 < 1/2 ... n = n: 1/n < ln n – ln (n – 1) < 1/(n – 1) 1 1 1 1 1 + + + < ln n <1 + + + n 2 n- 1 Cộng vế theo vế, ta được 2 3. Bài toán 4. Cho phương trình: ax² + x + c = 0 (a khác 0). Biết 6a + 3b + 2c = 0. Chứng minh rằng phương trình có ít nhất một nghiệm trong khoảng (0; 3). Hướng dẫn giải 1 1 f (x) = x 3 + bx + cx 3 2 Xét hàm số. Đạo hàm f’(x) = ax² + bx + c Áp dung Lagrange trên đoạn [0; 3] $x Î (0;3) :. f (3) - f (0) = f '(x) = 6a + 3b + 2c = 0 (3 - 0). Vậy phương trình: ax² + bx + c = 0 (a ≠ 0) có ít nhất một nghiệm trong (0; 3)..

<span class='text_page_counter'>(8)</span>

×